Questions tagged [lower-bounds]

The tag has no usage guidance.

Filter by
Sorted by
Tagged with
1 vote
1 answer
207 views

Markov Inequality for lower bounds

In a paper I found a strange application of Markovs inequality which I couldn't follow maybe you can help. $X_k$ is the set of $k$-element Subsets of $\mathbb{Z}_d^n$ we fix a $C^{-1} \in X_{k-1}$ and ...
David Trajkovski's user avatar
0 votes
0 answers
96 views

What is known about the average growth rate of the denominators of $n$ Egyptian fractions summing to one?

Motivation In the following question posted here on MO and over at MSE, user Noah Schweber asks about a weighted count on Egyptian fraction representations (EFRs). To that end, he defines the ...
Max Muller's user avatar
  • 4,226
0 votes
0 answers
40 views

Lower bound for the fractional Sobolev norm of the Hermites function

For $r \in (0, 2)$, I am interested in a lower bound for the quantity : $$I_r(n) := \int_{\mathbb{R}} |f_n(x)|^2 |x|^{r} dx$$ where $f_n(x) = (-1)^n (\sqrt{\pi} n! 2^n)^{-1/2} e^{x^2/2} \dfrac{d^n}{dx^...
jvc's user avatar
  • 183
0 votes
0 answers
45 views

Classifier-specific lower bounds on the misclassification rate in binary classification

Consider a binary classification problem for $(X,Y)$, and let $\hat{f}$ be a proposed classifier. We wish to bound the misclassification rate $P(\hat{f}(X)\ne Y)$. There are many known lower bounds on ...
tim523's user avatar
  • 13
2 votes
1 answer
213 views

Simple anticoncentration bound for binomially distributed variable

The following question, which arose during my research, seems deceivingly simple to me, but I could not find any elegant and formal argument. For a binomially distributed variable $X \sim \text{Bin} \...
reservoir's user avatar
0 votes
0 answers
207 views

Is it possible to bound Mertens function $M(n)$ from an inclusion-exclusion formulation?

In this post I proposed a formulation of Mertens function $M(n)$ using the inclusion-exclusion principle, as follows: $$M(n)=-\pi\left(n\right)+\left(\sum_{p_{i}<\sqrt{n}}\pi\left(\lfloor\frac{n}{...
Juan Moreno's user avatar
2 votes
1 answer
198 views

The lower bound of bivariate normal distribution

Suppose $(Z_1, Z_2)$ is the zero-mean bivariate normal distribution with covariance $\left( \begin{matrix} 1 & \rho; \\ \rho & 1\end{matrix} \right)$ with positive $\rho > 0$. What I want ...
香结丁's user avatar
  • 321
1 vote
0 answers
45 views

$\min(|\lambda_{\min}(A(c))|)$ for a special matrix $A(c)$ defined over $\{-1,-1\}^N$ [closed]

For a given constant $E$, is there way to find the lower bound of the following expression? $\min_{c\in\{-1,+1\}^N, -\sum_{i,j}c_ic_j=E}(|\lambda_{\min}(A(c))|)$ for matrix $A(c)$ defined over $\{-1,-...
tony's user avatar
  • 333
0 votes
0 answers
46 views

A Pinching inequality with respect to a continuous basis

For a generic trace-class matrix $A=\sum_{n,m}A_{n,m}|n\rangle\langle m|$, one can easily find the bound $\|P[A]\|_1\leq\|A\|_1\leq \|A\|_{1,1}$, where $\|A\|_1=\sum_{k}\sigma_k(A)$ is the trace norm, ...
La buba's user avatar
  • 101
3 votes
1 answer
206 views

Bounds for the crossing number in terms of the braid index?

Is there a lower bound on the crossing number of a knot (resp., link) with braid index $b$? For knots, I believe the smallest crossing number for braid index 2 is 3, the smallest crossing number for ...
Charles's user avatar
  • 8,914
3 votes
0 answers
62 views

Does this information theoretical thought experiment have a name or corresponding area of research?

I came up with the following thought experiment in my research in order to better understand the way Turing machines can transfer information through their tapes (the motivation is detailed below, isn'...
exfret's user avatar
  • 479
2 votes
1 answer
298 views

Lower bound on sum of independent heavy-tailed random variables

I have a sum of $n$ i.i.d random variables $X_i$ such that $E[X_i] = 0$,$\mathrm{E}[|X_i|^{1 + \delta}]$ exists for some $0 < \delta < 1$ but $\mathrm{E}[|X_i|^{1 + \delta+ \epsilon}]$ does not ...
Kaiyue Wen's user avatar
4 votes
1 answer
140 views

Existence of copula bound pointwise strictly smaller than the Fréchet-Hoeffding upper bound

Consider bivariate copulas $C_1$ and $C_2$ with $\max\{C_1(u,v), C_2(u,v)\}< M_2(u,v)$ for all $u,v \in(0,1)$, where $M_2(u,v) := \min\{u,v\}$ is the Fréchet-Hoeffding upper bound. Is there a ...
Corram's user avatar
  • 143
4 votes
0 answers
112 views

Log of a truncated binomial

Let $X$ follow a binomial distribution with $n$ trials and success probability $p$, and let $0\leq k\leq n$. Are there any natural approximations or bounds for the ratio $$\frac{\boldsymbol{E}\log\...
Tom Solberg's user avatar
  • 3,868
1 vote
1 answer
195 views

Tree width and clique width of regular graphs

Consider a $k$ regular graph of $n$ vertices, where $3 \leq k \leq (n-1)$. Is there any upper or lower bound, in the worst case, known for either the tree-width or the clique width of each $k$ regular ...
RandomMatrices's user avatar
0 votes
0 answers
163 views

How many elements have a "small" order in a finite field?

I'm hoping that this is an easy question for someone. How many elements can we expect to have multiplicative order at most $n^{1/c}$ in one of the finite fields $\mathbb{F}_p$ with $p$ prime with $n \...
Matt Groff's user avatar
2 votes
1 answer
88 views

Lower bound on the number of balanced graphs

Let $\alpha>1$ be a constant and define $B_n$ as the number of (labeled) balanced graphs with $n$ vertices and $\left\lceil \alpha n\right\rceil $ edges. The paper Strongly Balanced Graphs and ...
35T41's user avatar
  • 123
1 vote
0 answers
254 views

Lower bound on the sum of the product of random variables

Let $X_i$ be the $i$-th element of the vector $X=(X_1, ..., X_m)$ of i.i.d. random variables. I am looking for a lower bound for the expression $\mathbb{P}((\sum^n_{i=1}\prod^{m_i}_{j=1}(X_j))^2 \geq ...
Scriddie's user avatar
  • 129
1 vote
1 answer
349 views

Lower bound and limit of a sum with binomial coefficients

Let $$A_k = \sum_{i=1}^k i {3k-2i-1 \choose i-1} {2i-2 \choose k-i}$$ $$B_k = \sum_{i=1}^k i {3k-2i-2 \choose i-1} {2i-1 \choose k-i}$$ $$C_k = \sum_{i=1}^k (3k-2i-2) {3k-2i-3 \choose i-1} {2i\...
macat's user avatar
  • 125
5 votes
4 answers
780 views

Limit of a sum with binomial coefficients

Let $$A_k = \frac{\sum_{i=1}^ki{2k-i-1 \choose i-1}{i-1 \choose k-i}}{k{2k-1\choose k}}$$ $$B_k = \frac{\sum_{i=1}^ki{2k-i-2 \choose i-1}{i \choose k-i}}{k{2k-1\choose k}}$$ $$C_k = \frac{\sum_{i=1}^k(...
macat's user avatar
  • 125
4 votes
2 answers
244 views

An inequality involving binomial coefficients and the powers of two

I came across the following inequality, which should hold for any integer $k\geq 1$: $$\sum_{j=0}^{k-1}\frac{(-1)^{j}2^{k-1-j}\binom{k}{j}(k-j)}{2k+1-j}\leq \frac{1}{3}.$$ I have been struggling with ...
macat's user avatar
  • 125
2 votes
0 answers
151 views

When does Le Cam's method give tight lower bounds for distribution testing?

In the context of statistical estimation or distribution testing, Le Cam's method is a way to prove lower bounds on the minimax sample complexity ([1,2,3,4], further details below). My question is: ...
π314's user avatar
  • 33
2 votes
0 answers
134 views

How is the Cauchy-Schwarz equality and the assumption on the support of $g$ used to derive this bound?

I am currently reading On Uniqueness Properties of Solutions of Schrödinger Equations and a having trouble understanding a claim made on page 1819. Context from the paper: let $g\in C^\infty_0(\...
Dispersion's user avatar
2 votes
0 answers
240 views

Extension of the Gershgorin circle theorem for symmetric matrices and localization of positive eigenvalues

In mathematics, the Gershgorin circle theorem can be used to localize eigenvalues of a matrix (including symmetric). Let $A$ be a real symmetry $n × n$ matrix, with entries $a_{ij}$. For $i∈{1,…,n}$ ...
dtn's user avatar
  • 145
3 votes
0 answers
88 views

Probability of winning a $k$-rounds coin toss game

Let $p,q \in [0,1]$ with $p>q$. I denote by $B_k(p), B_k(q)$ two independent random variables following the binomial distribution, with parameters $(k,p)$ and $(k,q)$ respectively. Informal ...
Argemione's user avatar
  • 131
2 votes
1 answer
76 views

Asymptotic behavior of the moments of non-negative sequences

We consider a sequence $u = (u_k)_{k\geq 1}$ such that $u_k \geq 0$ for any $k \geq 1$. We assume that there exists a critical $p_c \in \mathbb{R}$ such that, for any $q<p_c <p$, $$\sum_{k=1}^\...
Goulifet's user avatar
  • 2,152
1 vote
0 answers
177 views

"Tails" of a multinomial distribution

Let $X_1,\dots,X_N$ denote a collection of independent samples of a uniform multinomial random variable in $\mathbb{Z}^k$, with the number of trials equal to $n\ll k$. (By "uniform", I mean ...
Tom Solberg's user avatar
  • 3,868
1 vote
0 answers
32 views

Are there known bounds on these ratios of chromatic polynomials?

The chromatic polynomial $P(G, \lambda)$ gives the number of proper vertex colorings of the graph $G$ with $\lambda$ colors. I'm interested in how many possible colorings you loose when you add an ...
M. Stern's user avatar
  • 111
2 votes
0 answers
172 views

Gershgorin-type bounds for smallest eigenvalue of positive-definite matrix

I would like to know if there are known results for bounding eigenvalues of positive-definite matrices, in particular gram matrices $AA^\top$ based on easily computable functions of $A$. Gershgorin ...
trenta3's user avatar
  • 109
2 votes
0 answers
143 views

Size of an “average” ϵ-net on the unit sphere

This is a question I originally asked on math.stackexchange, but didn't receive a satisfying answer. Let $\epsilon>0$ and consider constructing a set $S_\epsilon\subseteq S^{d-1}$ of points on the ...
R B's user avatar
  • 608
0 votes
0 answers
222 views

Lower-bound smallest eigenvalue of covariance matrix of $y = f(Ax)$, for $x$ uniform on unit-sphere

Let $A=(a_1,\ldots,a_)$ be a fixed $k \times d$ matrix (with $d$ large), and $x$ be a random vector uniformly distributed on the unit-sphere in $\mathbb R^d$. Let $f:\mathbb R \to \mathbb R$ be a ...
dohmatob's user avatar
  • 6,586
1 vote
0 answers
45 views

Analytic lower-bound for minimal value of $\|x\|^2$ such that $\|Cx-b\|^2 \le c^2$ (a hyperellipsoid)

Let $C$ be an $n \times p$ matrix and $b$ be a column vector of length $n$, and $c>0$. Let $E := \{x \in \mathbb R^p \mid \|Cx-b\| \le c\}$, a hyperellipsoid in nonstandard position. Question 1. ...
dohmatob's user avatar
  • 6,586
11 votes
3 answers
1k views

What is the limit of $a (n + 1) / a (n)$?

Let $a(n) = f(n,n)$ where $f(m,n) = 1$ if $m < 2 $ or $ n < 2$ and $f(m,n) = f(m-1,n-1) + f(m-1,n-2) + 2 f(m-2,n-1)$ otherwise. What is the limit of $a(n + 1) / a (n)$? $(2.71...)$
José María Grau Ribas's user avatar
1 vote
0 answers
28 views

Given a unit vector $x\in\mathbb R^d$, what is the worst possible within-cluster sum of squares for 2-means clustering?

This is a question I originally posted to math.stackexchange.com but it didn't attract any answers, and I was wondering if someone here can help. Consider a unit vector $x\in\mathbb R^d$ ($\|x\|_2=1$)...
M A's user avatar
  • 127
5 votes
2 answers
194 views

Distance of low-rank matrices to the identity for the $\infty$-norm

I am trying to get a lower bound (or even the exact value) of $$ \min_{X \in \mathbb{R}^{n\times n}} \|X - I_n\|_{\infty} \enspace \text{s.t.} \enspace \mbox{Rank}(X) = m $$ where $m \leq n$, and the ...
PAb's user avatar
  • 187
0 votes
0 answers
312 views

Lower-bound on expected value of norm of transformation of random vector with iid Rademacher coordinates

Let $n$ be a large positive integer. Let $A$ be a positive-definite matrix such with eigenvalues $\lambda_1 \ge \lambda_2 \ge \ldots \ge \lambda_n$ such that $\lambda_n = o(1) \to 0$ and $\lambda_i=\...
dohmatob's user avatar
  • 6,586
0 votes
0 answers
118 views

Spread of a disease on a modular chessboard (torus) - lower bound

I learned about the following result from one of Peter Winkler's books: It is impossible to infect the entire $n\times n$ chessboard (usual chessboard) starting from fewer than $n$ infected cells. The ...
Steve's user avatar
  • 1
2 votes
0 answers
321 views

Mertens Bound and the Riemann Hypothesis

Let $M(x)$ denote the Mertens function ($M(x)=\sum_{i=1}^{x}\mu(i)$ where $\mu(i)$ is the Möbius function) and let $\Lambda(i)$ denote the Mangoldt function ($\Lambda(i)$ equals $\log(p)$ if $i=p^{m}$ ...
Sourangshu Ghosh's user avatar
8 votes
0 answers
262 views

Restricted divisor summatory function

I have a problem that boils down to prove that the succession $\{a_n\}$ tends to infinity, where $$a_n:=1+\sum_{0\leq j<n}D_{2j+1}(n-j)$$ and $D_{m}(n)$ is the number of divisors $d>1$ of $n$ ...
Nick Belane's user avatar
2 votes
1 answer
437 views

Is this lower bound on the singular values of the sum of two matrices correct?

Equation 7 of this paper (Ramazan Türkmen, Zübeyde Ulukök, Inequalities for Singular Values of Positive Semidefinite Block Matrices, International Mathematical Forum, Vol. 6, 2011, no. 31, 1535 - 1545)...
Gabriele Oliva's user avatar
1 vote
0 answers
116 views

A lower-bound on matrix-function with vector product

I am currently trying to show that a sequence of homeomorphisms converges to some limiting homeomorphism using Anderson's the inductive convergence criterion. However I can't explicitly compute the ...
ABIM's user avatar
  • 5,001
0 votes
0 answers
220 views

Lower bound of exponential sum

This question is a close cousin of the following: Lower bound on exponential sums Let $\phi:[0,1]\to \mathbb R$ be a smooth function with $\frac 1 {10}<\phi'< 10, \frac 1 {10}<\phi''< 10$. ...
Thomas Yang's user avatar
2 votes
1 answer
1k views

Finding the expectation $\mathrm{E} (1/ X)$ for a negative binomial random variable $X$

Suppose a random variable $X$ is distributed as $\operatorname{NB}(\mu, \theta)$, and its mass is as follows $$ \mathrm{P}(X = y) = \binom{y + \theta - 1}{y} \left(\frac{\mu}{\mu + \theta}\right)^{y}\...
香结丁's user avatar
  • 321
2 votes
0 answers
583 views

An interesting sequence of numbers arising from the Riemann hypothesis

A very good coincidence occurred today with me. While just plotting random functions in Mathematica, I entered this command: ...
user avatar
0 votes
1 answer
275 views

better lower (and upper) bound for $i$'s moment of function of binomial random variable with $i = \frac{1}{j}, j \in \mathbb{N}$

I want to derive a lower bound for $E\left[\left(\frac{X}{k-X}\right)^{i}\right] $ with $X \sim Bin_{(k-1),p}$ and $ k \in \mathbb{N} $. So far I could prove that \begin{equation} E\left[\frac{X}{k-X}\...
qwert's user avatar
  • 89
2 votes
1 answer
392 views

How to find upper and lower bound

Let $\Sigma \in S_{++}^n$ be a symmetric positive definite matrix with all diagonal entries equal to one. Let $U \in \mathbb{R}^{n \times k_1}$, $W \in \mathbb{R}^{n \times k_2}$, $\Lambda \in \mathbb{...
newbie's user avatar
  • 61
0 votes
1 answer
215 views

Lower-bound on smallest singular-value of rectangular random matrix

Let $X$ be a random $N \times n$ matrix with iid entries from $\mathcal N(0, 1)$ and with $n/N =: \lambda(N,n) \le \lambda_0$, for some $\lambda_0 \in (0, 1)$. That is, $X$ is genuinely rectangular (...
dohmatob's user avatar
  • 6,586
3 votes
1 answer
85 views

If $X \sim N(0,I_m)$, what is a necessary and sufficient condition on $u_m > 0$ such that $\lim\sup_{m\to \infty} P(\|X\|^2 \ge u_m|X_1|) = 1$

Let $m$ be a large positive integer and $X=(X_1,\ldots,X_m) \sim N(0,I_m)$. I wish to show that the squared norm of $X$ is much much bigger than the absolute value of any of the $X_j$'s. For example, ...
dohmatob's user avatar
  • 6,586
1 vote
2 answers
209 views

What is the approximation of $\log(|\zeta'(\frac{1}{2}+it)|)$ in Dirichlet polynomial if it is exists?

I have done some search many times on web to find any approximation of $\log|(\zeta'(s))|$ in Dirichlet polynomial but I didn't got it, Probably that $\log(|\zeta'(s)|$ dosn't have a Dirichlet ...
zeraoulia rafik's user avatar
0 votes
0 answers
87 views

Upper bounding the sum with hypergeometric and binomial probabilities

Could you please help me upper bound this tricky expression: $$P(A)=\sum_{i=0}^n{\left( 1 - \dfrac{\binom kq \binom {n-k}{i-q}}{\binom {n}{i}} \right)}^I \binom ni p^i {(1-p)}^{n-i}$$. So far I only ...
abs135's user avatar
  • 1